Solution to Problem 688 | Beam Deflection by Method of Superposition

Problem 688
Determine the midspan value of EIδ at the left end of the beam shown in Fig. P-688.
 

688-overhang-beam-uniform-load.jpg

 

Solution 688

 

Another Solution by Area-Moment Method